EE370 Old Exam1

Embed Size (px)

Citation preview

  • 8/3/2019 EE370 Old Exam1

    1/17

    1

    EE370/01 Examination No. 1 Fall 2008/09

    (50 minutes)

    Kuwait University

    Electrical Engineering Department

    Name :

    Student I. D. : .

    Signature : .

    Problem No. Grade

    1 25

    2 25

    3 25

    4 25

    Total 100

  • 8/3/2019 EE370 Old Exam1

    2/17

    2

    Problem 1 (25 points): Drive the expression for)(

    )()(

    sR

    sUsM in terms of )(sGi for the following

    system:

    Problem 2 (25 points): Can you approximate the step response)10()(

    )5.10()(

    2

    sbsass

    ssC as

    the step response of 2nd order system with5

    2damping ratio and 2 seconds settling time? If yes,

    find the values of a and b? Justify your answer.

    Problem 3 (25 points): Use the Routh Table to find the range of K that keeps the following

    feedback system stable.

    Problem 4 (25 points): (a) Find the steady state error, )( te , in terms of a and b for the

    following system:

    (b) Find the values of a and b that yield zero steady state error.

    +

    +

    _ _

    C(s)R(s)

    +

    _

    ++

    _

    ++

  • 8/3/2019 EE370 Old Exam1

    3/17

    3

    EE370/01 Examination No. 1 Fall 2006/07

    Kuwait University

    Electrical Engineering Department

    Name :

    Student I. D. : .

    Signature : .

    Problem No. Grade

    1 20

    2 20

    3 20

    4 20

    5 20

    Total 100

  • 8/3/2019 EE370 Old Exam1

    4/17

    4

    Problem 1 (20 points): Can you use the partial fraction expansion2

    43

    2

    2

    2

    1

    )2(211

    s

    k

    s

    k

    s

    sk

    s

    k

    to find the inverse Laplace transform of22

    2

    )2)(1(

    3

    ss

    sif:

    a) 01

    k b) 0

    3 k

    c) 32 kk

    Explain your answer.

    324

    044

    124

    0

    )24()44()24()(

    )1()2)(1()2()2(3

    431

    321

    4321

    32

    431321

    2

    4321

    3

    32

    2

    4

    2

    3

    2

    2

    2

    1

    2

    kkk

    kkk

    kkkk

    kk

    kkkskkkskkkkskk

    sksskssksks

    a) No, you cannot solve the equationsb) No, you cannot solve the equationsc) Yes, you can solve the equations.

  • 8/3/2019 EE370 Old Exam1

    5/17

    5

    Problem 2 (20 points):

    a) (10 points): Drive the expression of the output )(sC in terms of( ), ( ), ( ), ( ), ( )M s H s G s R s and D s for the following system:

    b)

    (10 points): What conditions on ( )M s and )(sG you will impose if you want to make thesteady state output )(tc equals zero when 0)( sR and )(sD is a step function.

    LHPtheinGMofrootstheallplus

    MdMG

    HG

    s

    d

    GM

    HGsCb

    DGM

    HGR

    GM

    GCHDMCRHDG

    HDGXCa

    sss

    1

    )0(0)0()0(1

    )0())0(1(

    1

    )1(lim)

    1

    )1(

    1

    )(

    )

    +

    +

    +

    +

    -

  • 8/3/2019 EE370 Old Exam1

    6/17

    6

    Problem 3 (20 points): Consider the following closed-loop system:

    Find the values of1

    k and2

    k to get 0.5 settling time.

    ionapproximatorderndensuretok

    kkw

    T

    kw

    kksks

    ks

    ss

    ksk

    ss

    ks

    sT

    n

    s

    n

    20

    145.02

    84

    22

    )10()2(

    1021

    102)(

    1

    2

    2

    2

    212

    2

    1

    2

    1

    2

    2

    1

    C(s)R(s)

    +

    _

  • 8/3/2019 EE370 Old Exam1

    7/17

    7

    Problem 4 (20 points): Use Routh-Hurwitz criterion to find the values of0

    c ,1

    c ,2

    c that will cause

    the polynomial01

    2

    2

    34)( cscscsssM to have two real roots in the LHP, one real root in

    the RHP, and one root on the j-axis.

    4s 1 2c 0c

    3s 1 1c 2

    s 12 cc 0c

    1s

    12

    01121

    cc

    ccccc

    0s 0c

    Make rows 2 equal zero:12

    cc and 00 c

    Form the polynomial from scsrows 133

    :

    Differentiate the polynomial:1

    23 cs

    You have 3 symmetrical roots, select 01

    c (one sign change): one root on the RHP and its

    symmetrical root on the LHP plus one root on jw-axis. Also, one root on the LHP since you have

    4 roots in total.

    Then 012 cc and 00 c

    4s 1 2c 0c

    3s 1 1c 2

    s 3 1c 1

    s 1

    11

    3

    2

    3

    3c

    cc

    0s 1c

  • 8/3/2019 EE370 Old Exam1

    8/17

    8

    Problem 5 (20 points): A unity negative feedback system has the open loop transfer function

    2

    1)(

    2

    21

    ss

    skksG . Determine the gains

    1k and

    2k that minimize the steady state error

    due to the step, ramp, or parabolic input.

    )(2

    )2(lim)(

    2

    11

    lim)()(1

    lim)(12

    23

    3

    0

    2

    21

    00 sRkskss

    sssR

    ss

    skk

    ssR

    sG

    ste sss

    Then 05.0 12 kk for stability3

    s 1 2k 2

    s 2 1k 1s 12 5.0 kk 0

    s 1k

  • 8/3/2019 EE370 Old Exam1

    9/17

    9

    EE370/01 Examination no. 1 Fall 2005/06

    Kuwait University

    Electrical Engineering Department

    Name :

    Student I.D. :.

    Problem no. Grade1 20

    2 20

    3 20

    4 20

    5 20

    Bonus 20

    Total 120

  • 8/3/2019 EE370 Old Exam1

    10/17

    10

    Problem 1 (20 points): Given the following differential equation:

    2

    2

    ( ) ( ) ( 0.01)3 2 ( ) ( 0.01).

    d y t d y t d u t y t u t

    dt dt dt

    Use Laplace transform to find ( )y t for a unit step input ( )u t and zero initial conditions.

    Problem 2 (20 points): Find the value of 0a that will cause the following system to have somepoles on the j-axis. Use Routh-Hurwitz criterion.

    Problem 3 (20 points):

    c) (10 points): Drive the expression of the error ( ) ( ) ( )E s R s C s in terms of( ), ( ), ( ), ( ), ( )M s H s G s R s and D s for the following stable system:

    d) (10 points): If ( )R s and ( )D s are unit step functions, find the relation between( )H s and ( )M s in order to get zero steady state error.

    +

    -

    +

    - + +

  • 8/3/2019 EE370 Old Exam1

    11/17

    11

    Problem 4 (20 points): Consider the following closed-loop system:

    Can you find the values of 1 0k and 2 0k to meet the following specifications:

    Closed-loop damping ratio 0.8 . Closed-loop settling time 10sT sec. Steady state error of 0.1 due to a unit step input.

    Write the reason if you cannot find 1k and 2k .

    Problem 5 (20 points):a) (16 points): Sketch the root locus for the following system.

    Show the following on the root locus:

    Breakaway and/or break-in points. Angles of departure and/or arrival.

    b) (4 points): Can you find 0k to stabilize the closed-loop system.

    Bonus Problem (20 points): Consider the following feedback system

    where 7 1a . Find the value of 0k that will minimize the steady state error due to a unitstep input for

    C(s)R(s)+ +

    - -

    +

    -

    +

    -

  • 8/3/2019 EE370 Old Exam1

    12/17

    12

    EE370/01 Examination no. 1 Spring 2006

    Kuwait University

    Electrical Engineering Department

    Name :

    Student I.D. :.

    Problem no. Grade

    1 25

    2 25

    3 25

    4 25

    Bonus 20

    Total 120

  • 8/3/2019 EE370 Old Exam1

    13/17

    13

    Problem 1 (25 points): Given the closed-loop transfer function2

    ( ) 3 4( ) :

    ( ) 12 12

    Y s sT s

    R s s s

    for the

    following system

    e) (10 points): Determine the closed-loop poles and zeros. Also, show if there is any dominantpole(s).

    f) (15 points): Find the transfer function ( )G s ?

    Problem 2 (25 points): Given the following system

    a) (10

    points): Determine the value of 0k that will minimize the steady state error

    due to a unit step input ( )r t .

    b) (15 points): Determine the value of 0k that will minimize the sensitivity of the error ( )E s to a change in the parameter a .

    Problem 3 (25 points): Given3 2

    ( ) 6 M s s s ks c , find the range of k in term of c so that all

    the roots of ( )M s have a negative real part. Use Routh-Hurwitz criterion.

    Problem 4 (25 points): Consider the system:

    Determine the values of , ,a b c and k to have a unit step response ( )y t with: steady state value of 2,

    maximum amplitude of 3 and settling time of 4 sec.

    +

    -

    2

    -

    k

    -

  • 8/3/2019 EE370 Old Exam1

    14/17

    14

    Bonus (20 points): Given the Laplace transform of

    ( ( 1))d f a t

    dt

    equals G(s), find the

    Laplace transform of ( )f t in term of G(s)?

    EE370/51 Examination No. 1 Spring 2009

    (50 minutes)

    Kuwait University

    Electrical Engineering Department

    Name (Arabic) :

    Student I. D. : .

    Signature : .

    Problem No. Grade

    1 25

    2 25

    3 25

    4 25

    Total 100

  • 8/3/2019 EE370 Old Exam1

    15/17

    15

    Problem 1 (25 points): For the following system, find )(sU ?

    ++

    +_ __

  • 8/3/2019 EE370 Old Exam1

    16/17

    16

    Problem 2 (25 points): Consider the following translational mechanical system. A 2 N force f(t)

    is applied for 0t , find the values of vf , M, and K such that the response x(t) is given by the

    plot shown below.

    0 0.5 20

    1

    1.1

    Response

    x(t)

    Step Response

    Time (sec)

    Amplitude

    fv

    K

    f (t)

    x (t)

  • 8/3/2019 EE370 Old Exam1

    17/17

    17

    Problem 3 (25 points): Use the Routh Table to find the range of that makes the followingfeedback system unstable.

    Problem 4 (25 points): (a: 15 points) Derive the expression for the error )()()( sCsRsE in term

    of )(sH for the following system:

    (b: 10 points) Find the value of )0( sH that minimizes the steady state error.

    C(s)R(s)+

    _

    +

    _

    +

    +